Đến nội dung

the unknown nội dung

Có 210 mục bởi the unknown (Tìm giới hạn từ 30-05-2020)



Sắp theo                Sắp xếp  

#631611 x^3-7y^3+3xy(x+y)-24y^2+3x-27y=14

Đã gửi bởi the unknown on 06-05-2016 - 19:56 trong Phương trình - hệ phương trình - bất phương trình

Em thử làm theo cách này ạ, nếu có sai xin các anh chị góp ý chân thành  :) :

Điều kiện: $x\leq 3,y\geq -4$

Ta có: $x^3-7y^3+3xy(x+y)-24y^2+3x-27y=14\Leftrightarrow (x+y)^3-(2y+2)^3+3(x-y-2)=0\Leftrightarrow (x-y-2)((x+y)^2+(x+y)(2y+2)+(2y+2)^2+3)= 0$

Mà $((x+y)^2+(x+y)(2y+2)+(2y+2)^2+3)> 0$ với mọi $x,y$ nên $x-y-2=0$ hay $y=x-2$ nên $-2\leq x\leq 3$. Từ đó thay vào phương trình thứ hai ta có:

        $x^3+x^2-4x-1=\sqrt{3-x}+\sqrt{x+2}\Leftrightarrow (x+2)(x+1)(x-2)= (\sqrt{3-x}-1)+(\sqrt{x+2}-2)\Leftrightarrow (x+2)(x+1)(x-2)=\frac{2-x}{\sqrt{3-x}+1}+\frac{x-2}{\sqrt{x+2}+2}\Leftrightarrow (x-2)((x+1)(x+2)+\frac{1}{1+\sqrt{3-x}}-\frac{1}{\sqrt{x+2}+2})$

Do đó $x=2$ hoặc $((x+1)(x+2)+\frac{1}{1+\sqrt{3-x}}-\frac{1}{\sqrt{x+2}+2})=0$.

Nếu $x=2$ thì $y=0$ (thỏa).

Nếu $((x+1)(x+2)+\frac{1}{1+\sqrt{3-x}}-\frac{1}{\sqrt{x+2}+2})=0$ $\Leftrightarrow (x+1)(x+2)+\frac{\sqrt{x+2}-1+2-\sqrt{3-x}}{(1+\sqrt{3-x})(2+\sqrt{x+2})}=0$

$\Leftrightarrow (x+1)(x+2)+\frac{1}{f(x)}(\frac{x+1}{1+\sqrt{x+2}}+\frac{x+1}{2+\sqrt{3-x}})=0\Leftrightarrow (x+1)(x+2+\frac{1}{f(x)}.q(x))=0$

( với $f(x)=(1+\sqrt{3-x})(2+\sqrt{x+2}),q(x)=\frac{1}{1+\sqrt{x+2}}+\frac{1}{2+\sqrt{x+2}}$)

Dễ thấy $f(x),q(x)>0$ và $x+2\geq 0\Rightarrow x+2+\frac{q(x)}{p(x)}> 0\Rightarrow x+1=0\Leftrightarrow x=-1$ nên $y=-3$.

Vậy hệ có hai nghiệm là $(2;0),(-1;-3)$




#704950 VN TST 2018

Đã gửi bởi the unknown on 05-04-2018 - 15:56 trong Thi HSG Quốc gia và Quốc tế

Giả sử chọn $k = 2016$, và có $2^{2016}+1$ dòng, trong số các dòng này chứa tất cả các chuỗi nhị phân độ dài $k$ và có đúng 2 chuỗi giống nhau, xét hai chuỗi giống nhau ở cột thứ $2017$, một chuỗi chứa 0 một chuỗi để trống, ở cột thứ $2018$ thì chuỗi để trống ở cột $2017$ chứa 0 còn chuỗi còn lại thì để trống. Khi đó thì không thể xóa đi dòng nào được đâu bạn ạ.

 

Bảng này không phải bảng đầy đủ nhé bạn, lấy dãy nhị phân giống 2016 cột đầu của hai hàng này và đằng sau cho hai số 1,1 là thấy rõ.




#704837 VN TST 2018

Đã gửi bởi the unknown on 03-04-2018 - 23:18 trong Thi HSG Quốc gia và Quốc tế

Bài 2:

Câu b đã là gợi ý cho câu a. Một bảng gồm $2^k$ xâu nhị phân độ dài là $k$ và phần còn lại chừa trống thỏa mãn điều kiện là bảng tối giản.

câu b ta sẽ chứng minh nếu $m>2^k$ thì ta sẽ xóa được một dòng sao cho bảng vẫn còn là tối giản. Điều này có nghĩa là chứng minh rằng tất cả chuỗi nhị phân sinh ra từ dòng bị xóa sẽ thu được bằng cách thêm vào $0,1$ ở các dòng khác. Ta chỉ quan tâm tới cách sinh ra các chuỗi nhị phân với $k$ cột chứ $0,1$. Bây giờ xét tập A gồm $2^k$ chuỗi nhị phân có độ dài $k$. Mỗi một bước ta chọn một dòng của bảng (chỉ $k$ cột chứa 0,1), và xóa đi tất cả những chuỗi nhị phân trong A có thể sinh ra được bằng cách thêm $0,1$ vào các ô còn trống của dòng này. Với một dòng mà ta không xóa được trong A bất kì một chuỗi nào thì có nghĩa tất cả các chuỗi nhị phân sinh ra từ dòng đó có thể được sinh ra bởi các dòng khác, tức là xóa đi dòng đó bảng vẫn tối giản. A chỉ có $2^k$ phần tử mà ta có $m>2^k$ dòng thì dĩ nhiên sẽ chọn được một dòng chẳng xóa đi phần tử nào trong tập A được.

 

Thực ra lúc mình thi thì mình nhớ rằng ý b) không có điều kiện "tất cả các cột còn lại đều trống". Thực ra điều kiện chỉ cần có đúng $k$ cột có chứa hai số $0,1$ là đủ rồi. Lời giải của bạn vẫn đúng về ý tưởng nhưng có một xíu cải tiến như sau. Vẫn xét hàng mà chúng ta có thể xóa được trong lập luận của bạn, nếu nó không có phần tử nào ở các cột còn lại thì có thể xóa nó đi. Nhưng nếu nó có, và giả sử là số $0$ chẳng hạn ở một cột nào đó ngoài $k$ cột này thì ta vẫn có thể chứng minh mọi dãy nhị phân sinh ra từ hàng này thì có thể sinh ra từ một hàng khác bởi nếu không ta có thể lập luận và tạo thêm một cột nữa có chứa cả hai số $0,1$ và ta có điều vô lý.




#631697 USA(J)MO 2016

Đã gửi bởi the unknown on 07-05-2016 - 06:13 trong Tài liệu - Đề thi

Bạn giải thích rõ hơn giúp mình đoạn cuối được không? :(

Ý của mình là khi đó ta có thể bỏ ra tối đa $2016$ phần tử từ tập $X=\left \{ 1,2,3,...,6048 \right \}$ tức là có tối đa $2016$ cặp trong $3024$ cặp $(1;6048),(2;6047),...,(3024;3025)$ có phần tử bị loại nên còn ít nhất $1008$ cặp chưa có phần tử nào bị loại. Mà tổng hai số trong mỗi cặp bất kì đều có tổng bằng $6049$ nên nếu ta lấy $2016$ phần tử từ $1008$ cặp này thì tổng $2016$ phần tử đó đúng bằng $1008.6049=N$ đó bạn.




#631169 USA(J)MO 2016

Đã gửi bởi the unknown on 04-05-2016 - 14:01 trong Tài liệu - Đề thi

 

 

Ngày 2. (20/04/2016)
Bài 4. Tìm, và chứng minh số nguyên $N$ bé nhất sao cho nếu bỏ đi $2016$ phần tử từ tập $\{1, 2, \cdots , N\}$, chúng ta vẫn có thể tìm ra $2016$ số nguyên phân biệt trong các số còn lại có tổng bằng $N$.

 

Xử lý bài 4 luôn  :D :

Giả sử ta bỏ đi $ 2016$ phần tử $1,2,3,...,2015,2016$ từ tập $A= \left \{ 1,2,...,N \right \}$ thì khi đó trong các phần tử còn lại có $2016$ phần tử có tổng bằng $N$ và tổng các phần tử này không nhỏ hơn $2017+2018+...+4032$ nên $N\geq 2017+2018+...+4032= \frac{2016.6049}{2}= 1008.6049$.

Ta sẽ chứng minh rằng $N=1008.6049$ là giá trị nhỏ nhất cần tìm. Thật vậy xét tâp $X=\left \{ 1,2,...,6048 \right \}$ là một tập con của $A$, ta sẽ chứng minh rằng nếu bỏ đi tối đa $2016$ phần tử ra khỏi tập $X$ thì sẽ có $2016$ phần tử thuộc tập $X$ có tổng bằng $N$. Chia các phần tử của tập $X$ thành $3024$ cặp $(1,6048);(2,6047);...;(3024,3025)$, khi đó tổng hai số bất kì trong mỗi cặp đều bằng $6049$. Nếu ta bỏ ra tối đa $2016$ phần tử của tập này thì sẽ tồn tại ít nhất $1008$ cặp có phần tử chưa bị loại. Khi đó ta chọn $2016$ phần tử của $1008$ cặp này và tổng của $2016$ số này bằng $N$ nên ta có đpcm




#643644 USA TSTST 2016

Đã gửi bởi the unknown on 04-07-2016 - 17:41 trong Thi HSG Quốc gia và Quốc tế

USA TSTST 2016

(USA Team Selection Test for the Selection Team 2016)

 

Ngày 1 (25/6/2016)

 

Bài 1: Cho hai đa thức $A=A(x,y)$ và $B=B(x,y)$ là hai đa thức hai biến với hệ số thực. Giả sử rằng $A(x,y)/B(x,y)$ là đa thức theo $x$ với vô số giá trị của $y$ và là đa thức theo $y$ với vô số giá trị của $x$. Chứng minh rằng $B | A$, tức là tồn tại một đa thức $C$ với hệ số thực thỏa $A=B.C$

 

Bài 2: Cho $ABC$ là một tam giác không đều với trực tâm $H$ và tâm ngoại tiếp $O$. Gọi $M,N$ là trung điểm của $AH$ và $BC$ tương ứng. Giả sử đường tròn $\gamma$ với đường kính $AH$ cắt $(ABC)$ tại $G\neq A$ và cắt đường thẳng $AN$ tại $Q\neq A$. Tiếp tuyến tại $G$ của $\gamma$ cắt đường thẳng $OM$ tại $P$. Chứng minh rằng giao điểm của hai đường tròn $(GNQ)$ và $(MBC)$ cắt nhau tại điểm $T$ nằm trên $PN$.

 

Bài 3: Có tồn tại hay không một đa thức khác hằng $Q(n)$ với hệ số nguyên thỏa mãn tính chất: Với mọi số nguyên dương $n>2$ thì các số:

$Q(0),Q(1),Q(2),...,Q(n-1)$

nhận không quá $0,499n$ số dư phân biệt theo modulo $n$.

 

Ngày 2 (27/6/2016)

Bài 4: Giả sử $n$ và $k$ là các số nguyên dương thỏa mãn:

$\underbrace{\varphi (\varphi (...\varphi }_{\text{k lần}}(n)...))=1$

Chứng minh rằng $n\leq 3^k$.

Trong đó kí hiệu $\varphi(n)$ là số các số nguyên dương trong tập $\left \{ 1,2,3,...,n \right \}$ mà nguyên tố cùng nhau với $n$.

 

Bài 5: Trong mặt phẳng tọa độ có hữu hạn các bức tường là các đoạn thẳng không giao nhau và không có đoạn nào song song với các trục tọa độ. Một chiếc xe ủi bắt đầu từ một điểm bất kì trên mặt phẳng và di chuyển theo hướng dương của trục hoành. Mỗi khi tông một bức tường, chiếc xe sẽ rẽ một góc vuông và đi xa ra khỏi bức tường đó. ( Vì vậy mà chiếc xe luôn chuyển động song song với các trục tọa độ).

Chứng minh rằng chiếc xe không thể tông cả hai mặt của tất cả các bức tường.

 

Bài 6: Một tam giác $ABC$ có tâm nội tiếp $I$ và đường tròn nội tiếp tiếp xúc với các cạnh $BC,CA,AB$ lần lượt tại các điểm $D,E,F$. Gọi $K$ là chân đường vuông góc hạ từ $D$ xuống $EF$. Giả sử rằng đường tròn ngoại tiếp tam giác $AIB$ gặp đường tròn nội tiếp tại hai điểm phân biệt $C_1,C_2$, và đường tròn ngoại tiếp tam giác $AIC$ gặp đường tròn nội tiếp tại hai điểm phân biệt $B_1,B_2$. Chứng minh rằng trục đẳng phương của $(BB_1B_2)$ và $(CC_1C_2)$ đi qua trung điểm $M$ của $DK$.

 

Nguồn

P.s




#648044 Trường hè Toán học Miền Nam.

Đã gửi bởi the unknown on 05-08-2016 - 13:59 trong Thi HSG cấp Tỉnh, Thành phố. Olympic 30-4. Đề thi và kiểm tra đội tuyển các cấp.

Ta viết lại đề như sau: Tìm số tự nhiên $N_0$ lớn nhất không thể biểu diễn được dưới dạng $6x+10y+15z$ trong đó $x,y,z$ là các số tự nhiên. Hay nói cách khác mọi số tự nhiên lớn hơn $N_0$ đều biểu diễn được dưới dạng $6x+15y+10z$ với $x,y,z$ là các số tự nhiên.

Để giải bài toán này, ta sử dụng bổ đề sau:

Định lí Sylvester: Cho $a,b$ là hai số nguyên dương nguyên tố cùng nhau. Khi đó $N_0=ab-a-b$ là số nguyên dương lớn nhất không biểu diễn được dưới dạng $ax+by$ với $x,y$ là các số tự nhiên.

Bằng định lí này ta sẽ chứng minh kết quả khác: Cho $a,b,c$ là ba số nguyên dương đôi một nguyên tố cùng nhau. Khi số $N_0=2abc-ab-bc-ca$ là số nguyên dương lớn nhất không thể biểu diễn được dưới dạng $abx+bcy+caz$ với $x,y,z$ là các số tự nhiên.

(1) Ta sẽ chứng minh $2abc-ab-bc-ca$ không biểu diễn được dưới dạng $abx+bcy+caz$. Thật vậy nếu tồn tại $x,y,z$ tự nhiên để:

$2abc-ab-bc-ca=abx+bcy+caz$

$\Leftrightarrow 2abc=ab(x+1)+bc(y+1)+ca(z+1)\Rightarrow b\mid ca(z+1)\Rightarrow b\mid z+1\Rightarrow z+1\geq b$

Tương tự ta được $x+1\geq c$ và $y+1\geq a$ và ta suy ra $ab(x+1)+bc(y+1)+ca(z+1)\geq 3abc>2abc$ vô lí.

(2) Ta sẽ chứng minh tồn tại $x,y,z$ để $m+2abc-ab-bc-ca+1=abx+bcy+caz$ với $m\geq 0$. Biến đổi đẳng thức tương đương với:

$b(ax+cy-ac+a+c-1)+acz=m+abc-ac-b+1\geq abc-ac-b+1$

Theo định lí Sylvester thì tồn tại $u,z$ tự nhiên để $bu+acz=m+abc-ac-b+1$. Hơn nữa do $u\geq 0$ nên cũng tồn tại $x,y$ tự nhiên để $ax+cy=u+ac-a-c+1$. Và từ đó dễ dàng suy ra tồn tại $x,y,z$ tự nhiên thỏa mãn. Vậy kết quả được chứng minh.

Với $a=2,b=3,c=5$ thì ta được $N_0=29$. Vậy $29$ là số lớn nhất thỏa mãn đề bài.




#620027 Topic: Các bài toán về tính chia hết

Đã gửi bởi the unknown on 13-03-2016 - 13:05 trong Số học

Tìm số tự nhiên $n>0$ nhỏ nhất biết: 

a)$2^{n}-1\vdots 1001$

b)$2^{n}-2\vdots 1001$




#648428 Topic: [LTDH] Mỗi ngày hai bất đẳng thức.

Đã gửi bởi the unknown on 07-08-2016 - 16:36 trong Bất đẳng thức và cực trị

Bài 2: Cho $x,y,z$ là các số thực dương thỏa mãn: $x(x+y+z)=3yz$. Chứng minh rằng: $(x+y)^3+(x+z)^3+3(x+y)(y+z)(z+x)\le 5(y+z)^3$

Ủng hộ bài đầu tiên  :D

Giả thiết bài toán có thể viết lại thành: $(x+y)(x+z)=4yz$.

Áp dụng bất đẳng thức Bunyakovsky, ta có:

$(x+y)(x+z)\geq (x+\sqrt{yz})^2\Rightarrow 4yz\geq (x+\sqrt{yz})^2\Rightarrow x\leq \sqrt{yz}$

 

Vậy ta có:

 

$(x+y)^3+(x+z)^3\leq (y+\sqrt{yz})^3+(z+\sqrt{yz})^3$

 

$=(\sqrt{y}+\sqrt{z})^3((\sqrt{y})^3+(\sqrt{z})^3)=(\sqrt{y}+\sqrt{z})^4(y-\sqrt{yz}+z)$

 

$=\frac{1}{4}(y+2\sqrt{yz}+z)^2(4y-4\sqrt{yz}+4z)\leq \frac{1}{4}(\frac{4y-4\sqrt{yz}+4z+2(y+2\sqrt{yz}+z)}{3})^3=2(y+z)^3$

 

Và ta cũng có: $3(x+y)(y+z)(z+x)=12yz(y+z)\leq 3(y+z)^3$. Do ta có bất đẳng thức quen thuộc $(y+z)^3\geq 4yz(y+z)\Leftrightarrow 3(y-z)^2(y+z)\geq 0$

Cộng hai bất đẳng thức trên và ta được $(x+z)^3+(x+y)^3+3(x+y)(y+z)(z+x)\leq 5(y+z)^3$.

Nên có đpcm.




#648558 Topic: [LTDH] Mỗi ngày hai bất đẳng thức.

Đã gửi bởi the unknown on 08-08-2016 - 12:34 trong Bất đẳng thức và cực trị

Có thể mở rộng bài toán: Tìm hằng số $k$ lớn nhất để bất đẳng thức sau đúng:

                           $a^2+b^2+c^2+kabc\geqslant 3+k$

Có thể chọn $a=b=\frac{3}{2}$, $c=0$ thì ta được $k\leq \frac{3}{2}$. Ta sẽ chứng minh $k=\frac{3}{2}$ là hằng số tốt nhất cần tìm. Tức là ta chứng minh $2(a^2+b^2+c^2)+3abc\geq 9$.

Đặt $q=ab+bc+ca,r=abc,p=a+b+c=3$, bất đẳng thức tương đương $9+3r-4q\geq 0$.

Xét hai trường hợp:

$\text{TH 1:}$ $q\leq \frac{9}{4}$. Ta có: $9+3r-4q\geq 9-4q\geq 0$. (đúng)

$\text{TH 2:}$ $q\geq \frac{9}{4}$. Áp dụng bất đẳng thức Schur ta có:

$9+3r-4q\geq 9+3\frac{p(4q-p^2)}{9}-4q=9+(4q-9)-4q=0$.

Vậy với cả hai trường hợp bất đẳng thức đúng. Vậy bài toán được chứng minh.




#635711 Topic về phương trình và hệ phương trình

Đã gửi bởi the unknown on 26-05-2016 - 17:54 trong Phương trình - hệ phương trình - bất phương trình

 

Bài 444: 1)$x^2-x+1-\sqrt{2x-1}=0 $

2)$\frac{3x}{\sqrt{x-1}}=4+\frac{x}{\sqrt{x^2-3x+3}} $

3)$x^4+x^2+6x+9=(x^3+x^2+3x)\sqrt{x+3} $

4)$\frac{1}{\sqrt{-x^2+x+1}}+\frac{1}{\sqrt{-x^2-x+1}}=\frac{2}{\sqrt{1-x^2}}$

 

1.( Câu này chắc là dễ nhất  :) ) .ĐK: $x\geq \frac{1}{2}$

PT $\Leftrightarrow (2x-1)^2+3=4\sqrt{2x-1}$. Đặt $t= \sqrt{2x-1}\Leftrightarrow t^4-4t+3=0\Leftrightarrow (t-1)^2(t^2+2t+3)=0\Leftrightarrow t=1\Leftrightarrow x=1$.

3. ĐK: nếu $x<0$ thì $VT<0<VP$ nên pt vô nghiệm. Vậy $x\geq 0$. Do $x+3\geq 0$ nên áp dụng bất đẳng thức $Cauchy$ ta có:

$x^4+x^2+6x+9=x^4+(x+3)^2\geq \frac{(x^2+x+3)^2}{2}\geq x\sqrt{x+3}(x^2+x+3)\Rightarrow x^2=x+3\Rightarrow x=\frac{1+\sqrt{13}}{2}$ (do $x\geq 0$ nên ta loại nghiệm $ x=\frac{1-\sqrt{13}}{2}$).

4. ĐK: $\frac{1-\sqrt{5}}{2}\leq x\leq \frac{-1+\sqrt{5}}{2}$.

Đặt $\sqrt{-x^2+x+1}=a,\sqrt{-x^2-x+1}=b$ ( $a,b\geq 0$ ). PT $\Leftrightarrow \frac{1}{a}+\frac{1}{b}= \frac{2\sqrt{2}}{\sqrt{a^2+b^2}}\Leftrightarrow (a^2+b^2)(a+b)^2=8a^2b^2$.

Áp dụng BĐT $Cauchy$ có $(a^2+b^2)(a+b)^2\geq 2ab.4ab=8a^2b^2\Rightarrow a=b\Rightarrow x=0$.




#639381 Topic về phương trình và hệ phương trình

Đã gửi bởi the unknown on 10-06-2016 - 17:09 trong Phương trình - hệ phương trình - bất phương trình

Bài 457: Giải hệ phương trình:

          $\left\{\begin{matrix} x=y(4-y)\\ y=z(4-z)\\ z=x(4-x)\\ \end{matrix}\right.$




#630983 Topic về Bất đẳng thức, cực trị THCS

Đã gửi bởi the unknown on 03-05-2016 - 16:06 trong Bất đẳng thức và cực trị

Bài 4: Chứng minh rằng với mọi số thực dương $a;b;c$ ta có:

             $\sum \frac{1}{a+b}\geq \sum \frac{a}{a^2+bc}$

 

Do tính đối xứng của $a,b,c$ nên ta giả sử $0\leq a\leq b\leq c$

Ta có $VT-VP=\sum \frac{(a-b)(a-c)}{a^2+bc}=\frac{(a-b)^2(a+b)(2c-a-b)}{(a^2+bc)(b^2+ac)}+\frac{(c-a)(c-b)}{c^2+ab}\geq 0$ nên ta có dpcm.




#631019 Topic về Bất đẳng thức, cực trị THCS

Đã gửi bởi the unknown on 03-05-2016 - 19:59 trong Bất đẳng thức và cực trị

Góp vui một câu  :D:

Cho $a;b;c$ là số đo ba cạnh của một tam giác có chu vi là $2$. Tìm giá trị nhỏ nhất của biểu thức $a^2+b^2+c^2+2abc$




#630969 Topic về Bất đẳng thức, cực trị THCS

Đã gửi bởi the unknown on 03-05-2016 - 12:54 trong Bất đẳng thức và cực trị

Mình cùng góp ý một bài:

Cho a,b,c là các số thực dương thỏa mãn a+b+c=1.Cmr:

$\sum \sqrt{a+b}\geq \frac{3\sqrt{3}}{2\sqrt{2}}.(1+\sum ab)$

Bổ đề: Cho $x,y,z$ dương thỏa $x+y+z=3$, khi đó:$\sqrt{x}+\sqrt{y}+\sqrt{z}\geq xy+yz+zx$

Chứng minh: Ta có $\sqrt{x}+\sqrt{x}+x^2\geq 3x$

Tương tự có: $\sqrt{y}+\sqrt{y}+y^2\geq 3y$

                     $\sqrt{z}+\sqrt{z}+z^2\geq 3z$

Nên $2\sum \sqrt{x}+\sum x^2\geq 3(x+y+z)=9=(x+y+z)^2$$\Rightarrow \sum \sqrt{x}\geq \sum xy$

Áp dụng ta có: $\sum \frac{3}{2}(a+b)= 3\Rightarrow \sqrt{\frac{3}{2}}\sum \sqrt{a+b}\geq \frac{9}{8}\sum (a+b)(b+c)$

Mà $\sum (a+b)(b+c)=(a+b+c)^2+ab+bc+ca=1+\sum ab$

Nên $\sum \sqrt{a+b}\geq \frac{3\sqrt{3}}{2\sqrt{2}}(1+\sum ab)$ $\Rightarrow Q.E.D$




#630982 Topic về Bất đẳng thức, cực trị THCS

Đã gửi bởi the unknown on 03-05-2016 - 16:00 trong Bất đẳng thức và cực trị

Bài 1: Cho a,b,c là các số thực dương thỏa mãn $\frac{1}{a+b+1}+\frac{1}{b+c+1}+\frac{1}{a+c+1}\geq 1$.Cmr:

$a+b+c\geq ab+bc+ca$

 

Giả thiết tương đương: $\sum \frac{a+b}{a+b+1}\leq 2\Leftrightarrow 2\geq \sum \frac{(a+b)^2}{(a+b)^2+a+b}\geq \frac{4(a+b+c)^2}{2(a+b+c)+2\sum (a^2+bc)}$

$\Rightarrow a+b+c+(a^2+b^2+c^2+ab+bc+ca)\geq (a+b+c)^2\Leftrightarrow a+b+c\geq ab+bc+ca$




#619873 Topic tổng hợp các bài toán về phương trình nghiệm nguyên.

Đã gửi bởi the unknown on 12-03-2016 - 14:20 trong Số học

Tìm nghiệm nguyên dương của phương trình: $a^2+b^2+c^2=2(ab+bc+ca)$




#636644 TOPIC luyện thi vào lớp 10 chuyên toán năm 2016 - 2017.

Đã gửi bởi the unknown on 29-05-2016 - 21:38 trong Tài liệu - Đề thi

Xin lỗi mình nhầm

A = $a+\frac{1}{a}$ =$\frac{a^2+1}{a}$

Vì A là số nguyên dương nên a >0 và $a^2+1 = ak$ (với k là số nguyên dương)

Xét pt bậc 2 $a^2-ak+1=0$

$\Delta=k^2-4$

Pt có nghiệm nên $k\geq2$ hoặc $k\leq-2$

$ a1 = \frac{k+ \sqrt{k^2-4}}{2}  $

$ a2 = \frac{k- \sqrt{k^2-4}}{2}  $

Đồng thời 2 nghiệm của pt là số dương nên

S>0 và P>0

=> k>0

Từ đó suy ra $k\geq2$

Sau đó xét từng giá trị a1 ,a2 sao cho không lớn hơn 2017

suy ra được $2\leq k \leq2017$

Như vậy có 2016 giá trị của k và 4032 giá trị của a thoả mãn đề bài

Mình nghĩ mình có sai sót chỗ nào đó rồi các bạn xem mình làm có đúng không

Một lưu ý cho các bạn là để ý rằng phương trình $a+\frac{1}{a}=2$ chỉ có một nghiệm duy nhất. Do đó số giá trị cần tìm chỉ là $4031$ thôi bạn nhé.




#631474 TOPIC luyện thi vào lớp 10 chuyên toán năm 2016 - 2017.

Đã gửi bởi the unknown on 05-05-2016 - 21:22 trong Tài liệu - Đề thi

Mở rộng bài toán ra tập hợp số thực thì có giải được không bạn?

Dĩ nhiên là được chứ bạn. Điều này cũng đúng với tập hợp số hữu tỉ vì khi đó $x=\frac{y^2+y-46}{25}$ nên nếu $y$ là số hữu tỉ thì $x$ cũng là số hữu tỉ.




#631456 TOPIC luyện thi vào lớp 10 chuyên toán năm 2016 - 2017.

Đã gửi bởi the unknown on 05-05-2016 - 21:01 trong Tài liệu - Đề thi

Góp một bài dễ thôi: Cho $a,b,c$ là các số thực dương thỏa $ab+bc+ca=3$. Chứng minh rằng:

                $\frac{1}{a^2+1}+\frac{1}{b^2+1}+\frac{1}{c^2+1}\geq \frac{3}{2}$




#631461 TOPIC luyện thi vào lớp 10 chuyên toán năm 2016 - 2017.

Đã gửi bởi the unknown on 05-05-2016 - 21:06 trong Tài liệu - Đề thi

Tiện đây "cống hiến" cho topic một bài toán luôn cho đỡ bị loãng topic:

Tìm x,y nguyên dương thoả mãn phương trình: $25x+46=y(y+1)$

Giả thiết tương đương $100x+185=(2y+1)^2$. Do đó $(2y+1)^2$ là số chính phương lẻ chia hết cho 5 nên $(2y+1)^2\equiv 25$ ( $mod$ $100$).

Mà $100x+185\equiv 85$ ( $mod$ $100$) nên vô lý.

Vậy không tồn tại $x,y$ thỏa mãn đề bài.




#636469 TOPIC luyện thi vào lớp 10 chuyên toán năm 2016 - 2017.

Đã gửi bởi the unknown on 29-05-2016 - 10:28 trong Tài liệu - Đề thi

Bài 1:

        a) Tìm các cặp số thực $(a,b)$ để phương trình $x^2+ax+b=0$ có hai nghiệm nguyên và $3a+b=8$. Với các cặp số thực $(a,b)$ tìm được, giải phương trình đã cho.

        b) Giải phương trình: $(6x-3)\sqrt{7-3x}+(15-6x)\sqrt{3x-2}=2\sqrt{-9x^2+27x-14}+11$

Bài 2:

        a) Tìm tất cả các cặp số nguyên dương $(x,y,z)$ để $3^{x}+5^{y}=z^3$.

        b) Chứng minh rằng phương trình: 

                                  $\frac{1}{x}+\frac{1}{y}+\frac{1}{z}=\frac{1}{2017}$

hữu hạn nghiệm nguyên dương.

        c) Có bao nhiêu số thực $a$ sao cho biểu thức $a+\frac{1} {a}$ là một số nguyên dương không lớn hơn $2017$.

Bài 3: Chứng minh bất đẳng thức với ba số thực dương $a,b,c$ thỏa $a+b+c=3$:

                                   $a^2b+b^2c+c^2a\geq \frac{9a^2b^2c^2}{1+2a^2b^2c^2}$

Bài 4: Cho tam giác $ABC$ có ba góc nhọn nội tiếp đường tròn $(O)$, $BC$ cố định, $A$ thay đổi trên đường tròn, $BE$ và $CF$ là các đường cao. Các tiếp tuyến với đường tròn $(O)$ tại $B$ và $C$ cắt nhau tại $S$, các đường thẳng $BC$ và $OS$ cắt nhau tại $M$.

         a) Chứng minh: $\frac{AB}{AE}=\frac{BS}{ME}$

         b) Giả sử $AS$ cắt $(O)$ tại điểm thứ hai $K$. Chứng minh rằng $MA.MK$ không đổi khi $A$ thay đổi trên đường tròn $(O)$.

Bài 5: Chứng minh rằng với hai số thực không âm $a$ và $b$, ta có bất đẳng thức:

                                        $[2a]+[2b]\geq [a]+[b]+[a+b]$

trong đó $[a]$ là kí hiệu của số nguyên lớn nhất không vượt quá a.

Spoiler




#631469 TOPIC luyện thi vào lớp 10 chuyên toán năm 2016 - 2017.

Đã gửi bởi the unknown on 05-05-2016 - 21:16 trong Tài liệu - Đề thi

Thêm một bài nữa cho vui: Giả sử $m,n,p$ ($n$ là hằng số, $m,p$ là các số thực thay đổi) là các số thực sao cho phương trình $x^4+mx^3+nx^2+px+1$ có bốn nghiệm thực đều âm ( không nhất thiết phân biệt). Tìm giá trị lớn nhất của $p$ theo $n$.




#631727 TOPIC luyện thi vào lớp 10 chuyên toán năm 2016 - 2017.

Đã gửi bởi the unknown on 07-05-2016 - 12:52 trong Tài liệu - Đề thi

Góp một bài khá là hay nha:
Chứng minh rằng với mọi $n\geq 6$ thì $a_{n}$ luôn là số chính phương với $a_{n}=1 + $ $\frac{2.6.10.....(4n-2)}{(n+5)(n+6)...(2n)}$

Có $\frac{2.6.10.....(4n-2)}{(n+5)(n+6)...(2n)}=2^n.\frac{1.3.5...(2n-1)}{(n+5)(n+6)...(2n)}=\frac{1.2.3....(2n-1)(2n)}{1.2.3...n.(n+5)(n+6)...(2n)}= (n+1)(n+2)(n+3)(n+4)$.

Vậy $a_{n}=1+(n+1)(n+2)(n+3)(n+4)=(n^2+5n+5)^2$ là một số chính phương.




#631758 TOPIC luyện thi vào lớp 10 chuyên toán năm 2016 - 2017.

Đã gửi bởi the unknown on 07-05-2016 - 16:24 trong Tài liệu - Đề thi

Kết quả thì ra đúng rồi, nhưng bạn xem lại phần chữ đỏ nha, không ổn cho lắm

Sao lại không ổn vậy bạn?